MD, PhD, MAE, FMedSci, FRSB, FRCP, FRCPEd.

Craniosacral therapy (CST), which, confusingly, is sometimes also called ‘cranial osteopathy’, was invented less than half a century ago by an osteopath. He thought that the spinal fluid is pulsating, the cranial bones are sufficiently movable to enable a therapist feel this pulse from the outside, and that it is possible to influence this process with very gentle manual manipulations which, in turn, would restore health in sick individuals. According to the inventor, the CST-practitioner uses his or her own hands to evaluate the craniosacral system by gently feeling various locations of the body to test for the ease of motion and rhythm of the   fluid pulsing around the brain and spinal cord. Soft-touch techniques are then used to release restrictions in any tissues influencing the craniosacral system.

But how does CST work? Let’s ask a practitioner who surely must know best:

When a self-development issue is linked to the illness, it is enough for that issue to be acknowledged by the client (without any further discussion unless the client desires it) for the body to release the memory of that issue – sensed by the therapist as tightness, tension, inertia within the body’s systems – so that the healing can proceed.

Several treatment sessions may still be needed, especially if the condition is a long lasting one. Our bodies’ self healing mechanisms rely on a combination of the various fluid systems of the body (blood and lymph flow and the fluid nature of the cells making up all the organs and systems within our bodies) and the body’s energy fields. Our hearts generate their own electrical signal independently of the control of our brains. Such signals travel around the body through the blood and other fluid systems. Blood is an excellent conductor of electricity and, when electricity flows through a conductor, magnetic fields are created. It is with these fields that the craniosacral therapist works.

These same fields store the memory of the events of our life – rather like the hard disk on a computer – but these memories can only be accessed when the underlying Body intelligence ‘decides’ it is needed as part of the healing process. There is absolutely no danger, therefore, of more being revealed than is strictly necessary to encourage the client back onto their self development route and to enable healing to take place.

To many desperate patients or distressed parents of ill children – CST is often advocated for children, particularly those suffering from cerebral palsy – this sort of lingo might sound impressive; to anyone understanding a bit of physiology, anatomy etc. it looks like pure nonsense. CST has therefore been considered by most independent experts to be on the lunatic fringe of alternative medicine.

Of course, this does not stop proponents to make and publicise big therapeutic claims for CST; it would be quite difficult to think of a condition that some CST-practitioner does not claim to cure or alleviate. One UK organisation boldly states that any symptom a patient may present with will improve in the hands of one of their members; in the eyes of its proponents, CST clearly is a panacea.

But, let’s be fair, the fact that it is implausible does not necessarily mean that CST is useless. The theory might be barmy and wrong, yet the treatment might still be effective via a different, as yet unknown mechanism. What we need to decide is evidence from clinical trials.

Recently, I have evaluated the findings from all randomised clinical trials of CST. I was pleasantly surprised to find that 6 such trials had been published, one would not normally expect so many studies of something that seems so utterly implausible. Far less impressive was the fact that the quality of the studies was, with the exception of one trial, deplorably poor.

The conditions treated in the trials were diverse: cerebral palsy, migraine, fibromyalgia and infant colic. All the badly-flawed studies reported positive results. The only rigorous trial was the one with children suffering from cerebral palsy – and here the findings were squarely negative. The conlusion of my review was blunt and straight forward: “the notion that CST is associated with more than non-specific effects is not based on evidence from rigorous randomised clinical trials“. This is a polite and scientific way of saying that CST is bogus.

Why should this matter? CST is popular, particularly for children. It is a very gentle technique, and some might argue that no harm [apart from the cost] can be done; on the contrary, the gentle touch  might even calm over-excited children and could thus be helpful. Who then cares that it has no specific therapeutic effects?

Few people would argue against the potential benefits of gentle touch or other non-specific effects. But we should realise that, for achieving them, we do not need CST or other placebo-treatments. An effective therapy that is given with compassion and empathy will do the same trick; and, in addition, it will also generate specific therapeutic effects.

What follows is simple: administering CST or other bogus treatments [by this, I mean a treatment for which claims are being made that are not supported by sound evidence] means preventing the patient from profiting from the most important element of any good treatment. In such cases, patients will not be treated adequately which can not just cost money but, in extreme cases, also lives.

In a nutshell: 1) ineffective therapies, such as CST, may seem harmless but, through their ineffectiveness, they constitute a serious threat to our health; 2) bogus treatments become bogus through the false claims which are being made for them; 3) seriously flawed studies can be worse than none at all: they generate false positive results and send us straight up the garden path.

162 Responses to Up the garden path: craniosacral therapy

  • LOL Yet again Prof Ernst feels everybody elses research is flawed yet sites his own systematic review with only one author, himself!!!!!!!BIAS
    Do we really have to go over the pitfalls of a single author systematic review again? Why do you constantly talk about flawed research then offer up more flawed research?

    In a nutshell, if you offer up a systematic review with only one author you can not rule out author bias and all the other chances of bias as described for cochrane reviews. Your study as sited carries no more weight than this blog.

    • Perhaps fedup would like to provide even the tiniest shred of evidence for the effectiveness of craniosacral therapy, for any condition?

      A striking feature of all alternative reality medicine is there are never any actual people who have actually been cured of anything. Never any real life examples where diagnoses, treatments and outcomes can be corroborated. If anyone had ever been cured of anything by any alternative reality medicine, the cured examples would be everywhere and easily confirmed. But they are never anywhere to be found. That’s the problem… no cases, no bodies.
      What we do have in abundance for every conceivable form of alternative reality therapy and medicine (no matter how incompatible with each other) are fantastical claims, anecdotes, delusions, fabrications and falsehoods. Never any actual evidence outside a statistical insignificance or error.

    • please read what i actually said: there were 6 studies; only one was of good methodology, and it arrived at a negative conclusion. no review can rule out author bias; the bronfort-review, for instance, had several authors (all chiros) and was very overtly biased. multiple authors can even reinforce bias. bias should be demonstrated, not assumed! where can you demonstrate bias in my single-author review?

    • fed up:
      Friday 26 March 2010 at 2:55 PM
      “The assumption is that well-trained scientists will retain their objectivity, whatever their enthusiasms.”
      lol

      The above comment was left under a post about Edzard on my blog. Note the date. You make a similar comment on a previous post on this blog. Time to change the record, fedup. Either present a coherent argument to support your view that Edzard is biased or STFU.

    • Why all the fuss about the review having a single author? Do you know anything about what happens in the peer-review process?

  • If you feel that way, fedup, surely you must ask Prof Ernst for evidence: his criteria & what they are based on.

    • If only there was somewhere that Professor Ernst had published his criteria and his assessment of the trials so that fedup could read them. Like, say, the systematic review linked from the blog post…

  • It doesn’t matter what title they give this ‘treatment’ (craniosacral, Cranial Osteopathy, Osteopathy in the Cranial Field) it is total nonsense. Nonsense is one thing (and the internet is full of it), but when nonsense is being touted as a treatment for some very serious/complex medical conditions …… it becomes dangerous nonsense!

    If people take the time to look at this treatment they will find that there is NO good quality evidence to support it as a treatment for even a single condition, let alone the huge range of conditions often claimed.

    The response from Fedup is typical of the vast majority of CAM therapists and supporters. They criticise, rant and complain, but routinely fail to supply any robust evidence to support the claims.
    When good quality research is done (with input & agreement from osteopaths) and it shows that there is NO benefit … they simply ignore it and carry on! (Did somebody mention ‘Bias’ !)

    I wait with baited breath for somebody to post some good quality evidence for CST that reports positive results !

    SB

  • LOL-LOL, but it should be clear that CST has wrong asumtions about the anatomy in humans -@fed up- do you have floating bones in your skull? Be careful 😉 . Normally skulls look like this one :skull: and yours?

  • I had CST on a regular basis for ages: it was an absolutely wonderful massage, though I’m not sure if it did anything else.

    • I have no doubt that CST given as a ‘massage’ can be both relaxing and enjoyable …. If only that was the extent of the claims attached to it!

      What tends to happen is people with absolutely no medical qualifications offer to use that relaxing ‘massage’ to treat, cure or prevent potentially serious conditions such as “colic, sleeplessness, irritability, ear infections and help more serious conditions such as cerebral palsy, autism and learning difficulties.” (These were taken straight from a current CST website).

      SB

  • Our child had serious issues in his/her first year and we went to an osteopath who gave him/her CST. All I know is that the issues were gone after the second treatment and our baby slept close to 24h after the first. I am not interested in evidence, I am interested in results. If this is placebo or anything else, I just dont care.

  • Pleps: so, let’s assume your child got better because of the natural history of the condition which means she would have got better anyway, even without any treatment at all. Don’t you think it is important to know why she improved? How else would we be able to advise future patients what to do when they go through a similar experience as yours? You say that you are interested in results, not evidence. I do not think one can seperate the two issues.

  • I am a licensed massage practitioner and I wish to thank you Edzard for this post/review. And you Skeptic Batista for your no nonsense responses.
    Massage is full of false claims and I hate it. It is so hard for me to prove it to my peers because i cant find much evidence to counter the claims (assuming they would even look) .
    It breaks my heart to have to tell my clients that, no, I can’t improve your circulation to your diabetic foot. No I can’t massage away your fat, or detox your liver/kidney, or do anything but help you loosen up by manipulating your muscles, making micro tears in your muscles (i assume that is why the muscle loosens up and they are sore the next day?) and relaxation … None of them accept what I say, and promptly find someone else who will tell AND SELL them what they want to hear.

    These claims are at their worst deadly, as they take precious time and money away from more effective treatments, and at best pernicious .. as they distract us from our real problems, with false solutions.
    Edzard if you could please do more reviews on massage claims I would be most appreciative. Does massage actually improve circulation? What’s all this nonsense about massaging away toxins (like lactic acid)? Just what is a “knot” anyway? Can you actually break up scar tissue with massage? These things and more are taught to us over and over in massage school and are the foundation for the required continuing education classes. THANK YOU SO MUCH! – Your loyal twitter follower from Washington, Justin .

  • @Edzard: After one year of getting worse and worse? Healing came exactly in these 2 weeks and the 24h sleep was a coincidence that began 10 minutes after the treatment in the car on the way home? After 11 months of not sleeping longer than 3 hours at a stretch?
    I really dont care about evidence. Healing is as much talent as it is knowledge imho.

  • Plebs: natural history was only one example of an explanation that is not directly related to the treatment. Placebo-effect is another. Do you really mean that we don’t need to know whether a treatment is more effective than a placebo?

  • @Phelps the thing about sleep is no matter how hard you fight it, you will lose eventually, we all do. What did you expect her to do? Did she sleep as a result of or inspite of the treatment? If the claim of cst was that it can make people sleepy I’m all for it. You lay still for an hr or more and someone gently touches your head, trust me it does not require training, as part of my class requirement I had to take the training. Its a joke. Take your fingers and lay them on your child’s head with ” no more pressure then the weight of a dime” she will fall asleep, maybe not then and there, maybe in a day, maybe even two days. I promise you she will fall asleep, and then when she does, call yourself a healer.

  • I mean we should know what treatment is most effective at all, Placebo, gods, spaghetti monsters etc. or anything else included. But if we dont know, the world will continue to turn.

  • Ok, I do agree with that: the world will continue to turn. It would also still turn, if we were blood-letting each other to death as we did ~200 years ago. I stand for progress; what do you stand for?

  • I stand for reason. Progress is nothing without it and lots of scientific findings lack reason. Look at the nuclear bomb or other WMDs. Great science without reason. What was the topic here?

  • Pleps, I think that’s a misuse of the word ‘reason’. It sounds as if you mean something more like ‘compassion’ or ‘humaneness’. If you stood for reason, you would want to know whether in fact CST actually helped your child or not and, if so, why. You’re assumption that she got better because she had the treatment is flawed reasoning. Post hoc ergo propter hoc.

  • @Skeptical_UK: I mean “reason” and sorry, but you dont have the determinative authority over the word. And sorry again, but I dont have to know everything (as stated before), often its enough to see it with my own eyes without a control group and dozens of young, healthy, non-smoker, male volunteers. Its cheaper as well …

  • Pleps: in healthcare, reason means aiming at improving things so that tomorrow we have better treatments than today. this cannot be achieved without evidence. your “reason” turns out to be unreason which leads us straight back to the dark ages.

  • That what I dislike about hardcore sceptics. They are fighting a religious war too. Thats your opinion about reason in healthcare. I cannot find a reliable source for your statement. And I dont let you “occupy” words for this war, at least I try like the englighteners against the catholic or anglican church …

    • Is it really “fighting” if people merely point out mistakes in your thinking? Most sceptics (if not all) would LOVE for those “alternative” treatments to work, but the best evidence usually tells us they don’t. Of course believing this evidence is easier when you always were sceptic about the “therapy”, but even if it is something you like or once believed in, if the evidence points the other way, you let go of it. That’s reasonable – insisting that your own experience trumps unbiased/less biased approaches isn’t, it’s foolish.
      I can’t help but think in this kind of conversation (A: “There’s no evidence that intervention X helps with Y.” – B: “But it worked for me, that’s all the evidence I need!”) that what people are really saying is “I paid good money for that so I need to believe X was worth it, or else I’ll feel like an idiot. I’m not an idiot, so either tell me that it works/might work or shut up.”

  • you cannot find evidence that reason improved medicine; well, take off your blinkers and look! it is you who uses terminology like hardcore, religious war, and it is you who hides behind a pseudonym. what does this tell us?

    • That tells you that I for sure doesnt want a group of people to know who I am because that could hurt my life, my family and persons I like.

  • Pleps,

    Thank you but there’s no need to apologise. It is quite clear from my post that I’m not claiming “determinative authority over the word”, nor did I suggest that you “have to know everything”.

    That you should have interpreted my comment in that way – on top of the logical fallacy you offer for defending CST – is further illustration that your reasoning isn’t as good as you think it is, so let’s leave it there.

  • Pleps, as you can see Prof Ernst has his own view of the world, he decides, of the 6 studies only 1 was good enough and guess what, that had a negative outcome so only furthered Prof Ernst view, forget the other 5 because in (in his opinion) the research was flawed. How? doesn’t matter. He then puts forward his research, a systematic review which is doomed to bias almost from the start if there is only 1 author (according to Cochrane rules).

    • fedup: would it not be more productive to engage with the arguments i have posted regarding my “biased” review than stubornly repeating your false notions?

  • to skepticat etc, I’m not defending cranial sacral, as I don’t know anything about it, I am merely pointing to the flaws in Prof Ernsts review. If you cannot see the bias in his work then your ability to “retain their objectivity, whatever their enthusiasms” isn’t as good as you think it is.

    • Nice try, fedup, and I particularly love the irony that you are doing exactly what you accuse Edzard of doing, which is asserting something is true without substantiation. I asked you for a coherent argument that Edzard is biased, not just repetition of your opinion that he is.

      To Pleps you said,

      “[Edzard] decides, of the 6 studies only 1 was good enough and guess what, that had a negative outcome so only furthered Prof Ernst view, forget the other 5 because in (in his opinion) the research was flawed. How? doesn’t matter.”

      You call this “merely pointing to the flaws in Prof Ernsts review” but what this comment reveals is that you have not followed the link he provides and actually read the review, which explains everything in detail, including what was wrong with each of the five rejected trials.

      Here’s a snippet:

      “This study was small, not blinded and further limited by the single-therapy session design. Other potential flaws included an inappropriate randomisation procedure, possible baseline differences between groups and lack of detail on the study methodology. Consequently, there was a high risk of bias, which might have produced a false negative result.”

      So it’s not just a matter of “his opinion”; anyone who is able to retain their objectivity would come to the same conclusion.

      If you actually want to try to engage intelligently on this subject instead just repeating your groundless ad hominem, I invite you to make a case for the inclusion of the five rejected trials in spite of the perceived flaws in them.

      • skepticat, Please enlighten me to the flaws in all the other pieces I have highlighted here and then let me know how the conclusion to a single author systematic review that is as liable to author bias and all the other types of bias as listed by the cochrane handbook, carries any more weight than the other studies? You blindly dismiss the other 4 studies plus all the rest available on pub med because of their flaws then accept totally a flawed piece by Prof Ernst. Thats irony.

        • I believe Prof ernst to be biased, but that does not detract from the fact that his research is flawed and as likely to have bias as any other flawed piece of research.
          And back to my initial statement.

          “Why do you constantly talk about flawed research then offer up more flawed research?”

        • It would be irony if were true, but it isn’t and you’re sounding more like a troll with every post.

          Nowhere have I “blindly dismissed” anything and, since you are the one claiming that EE is offering up research that is “totally flawed”, the onus is on you to say why it is flawed. All you’ve offered so far can be reduced to this:

          P1: To eliminate bias a review must have more than one author
          P2: E’s systematic review of CST has only one author.
          C: Therefore E’s systematic review of CST is “totally flawed”.

          This argument is unsound because the first premise is patently false. Edzard addressed this in his first response to you and put to you the obvious question, which you ignore because, I suspect, you don’t have a clue what you’re talking about. Prove me wrong! Here’s his question again:

          “where can you demonstrate bias in my single-author review?”

          Well?

          .

          • Skepticat, I don’t need to demonstrate bias, the fact that prof Ernst is the only author means he formulated the review question he designed the criteria, he decided the methodology, he decided which studies fitted his criteria,he evaluated the findings, as there was no other input from another author, editor etc how can you really believe that no bias was present in his review? especially when you look at his conclusion, or his interpretation of his own conclusion.

          • @fedup:

            Yes, you do need to demonstrate bias, because you are claiming that Professor Ernst’s review is biased.

            You need to demonstrate bias, or withdraw the claim.

          • @fedup

            You also need to demonstrate that Professor Ernst’s research is flawed. Bare assertions will not do.

  • “CONCLUSIONS:
    A series of treatments using osteopathy in the cranial field, myofascial release, or both improved motor function in children with moderate to severe spastic cerebral palsy. These results can be used to guide future research into the effectiveness of OMT or acupuncture in treating children with spastic cerebral palsy.”

    Rubbish research eh Prof.

    “AUTHORS’ CONCLUSIONS:
    The studies included in this meta-analysis were generally small and methodologically prone to bias, which makes it impossible to arrive at a definitive conclusion about the effectiveness of manipulative therapies for infantile colic.The majority of the included trials appeared to indicate that the parents of infants receiving manipulative therapies reported fewer hours crying per day than parents whose infants did not, based on contemporaneous crying diaries, and this difference was statistically significant. The trials also indicate that a greater proportion of those parents reported improvements that were clinically significant.”

    Again 6 authors for this study but hey, bias to high, right Prof.

    “Cranial osteopathic manipulative medicine’s growing evidence base.” on pub med, but obviously bogus.

    But come on it can’t work because it doesnt actually do anything does it prof?

    CONTEXT:
    The use of cranial osteopathic manipulative medicine (OMM) to alter cerebral tissue oxygen saturation could play a role in the maintenance of cerebral homeostasis.
    OBJECTIVE:
    To examine the effects of cranial OMM on cerebral tissue oxygen saturation (S(CT)O(2)) and cardiac autonomic function in healthy adults.
    METHODS:
    Cranial OMM augmentation and suppression techniques and sham therapy were randomly applied to healthy adults. During cranial OMM and sham therapy, S(CT)O(2) of the prefrontal cortex was determined bilaterally by using near-infrared spectroscopy. Heart rate, blood pressure, and systemic arterial blood oxygen saturation (SaO(2)) were also measured. Power spectral analysis was applied to continuous 4-minute R-R intervals. Measurements were made during 2-minute baseline periods, during 4-minute applications of the techniques, and during 5-minute recovery periods.
    RESULTS:
    Twenty-one adults (age range, 23-32 y) participated in the present study. Differences in mean baseline measurements for the augmentation technique, suppression technique, and sham therapy were not statistically significant for heart rate, blood pressure, SaO(2), left S(CT)O(2), or right S(CT)O(2). During the suppression technique, there was a statistically significant decrease in both left (slope [standard deviation]= -0.33 [0.08] %/min, R(2)=0.85, P=.026) and right (slope [standard deviation]=-0.37 [0.06] %/min, R(2)=0.94, P=.007) S(CT)O(2) with increased cranial OMM time. However, neither the augmentation technique nor the sham therapy had a statistically significant effect on S(CT)O(2). Decreases in normalized low-frequency power of R-R interval variability and enhancements of its high-frequency power were statistically significant (P=.05) during cranial OMM and sham therapy, indicating a decrease in cardiac sympathetic influence and an enhanced parasympathetic modulation.
    CONCLUSION:
    The cranial OMM suppression technique effectively and progressively reduced S(CT)O(2) in both prefrontal lobes with the treatment time.

  • Theres more rubbish research out there!

    Exploring the impact of osteopathic treatment on cranial asymmetries associated with nonsynostotic plagiocephaly in infants.
    CONCLUSIONS:
    These clinical findings support the hypothesis that osteopathic treatments contribute to the improvement of cranial asymmetries in infants younger than 6.5 months old presenting with NSOP characteristics.

    Well, more poor research?

    Effect of osteopathy in the cranial field on visual function–a pilot study.
    STUDY DESIGN:
    Randomized controlled double-blinded pilot clinical trial.
    SUBJECTS:
    Adult volunteers between ages 18 and 35 years who were free of strabismus or active ocular or systemic disease were recruited. Inclusion criteria were refractive error ranging between six diopters of myopia and five diopters of hyperopia, regular astigmatism of any amount, and cranial somatic dysfunction.
    INTERVENTION:
    All subjects were randomly assigned to the treatment or control group. The treatment group received a single intervention of osteopathy in the cranial field to correct cranial dysfunction. The control group received light pressure of a few ounces of force applied to the cranium without osteopathic manipulative treatment.
    MEASUREMENTS:
    Preintervention and postintervention optometric examinations consisted of distant visual acuity testing, Donder push-up (ie, accommodative system) testing, local stereoacuity testing, pupillary size measurements, and vergence system (ie, cover test with prism neutralization, near point of convergence) testing. Global stereoacuity testing and retinoscopy were performed only in preintervention to determine whether subjects met inclusion criteria. Analysis of variance (ANOVA) was performed for all ocular measures.
    RESULTS:
    Twenty-nine subjects completed the trial-15 in the treatment group and 14 in the control group. A hierarchical ANOVA revealed statistically significant effects within the treatment group and within the control group (P <.05) in distance visual acuity of the right eye (OD) and left eye (OS), local stereoacuity, pupillary size measured under dim illumination OD and OS, and near point of convergence break and recovery. For the treatment group vs the control group, a statistically significant effect was observed in pupillary size measured under bright illumination OS (P <.05).
    CONCLUSIONS:
    The present study suggests that osteopathy in the cranial field may result in beneficial effects on visual function in adults with cranial asymmetry. However, this finding requires additional investigation with a larger sample size and longer intervention and follow-up periods. (ClinicalTrials.gov number NCT005105

  • thank you for making my point: the existing RCTs are methodologically weak, e.g. small sample size [one could be as biased as possible, anyone who can count must confirm this; lack of independent replications etc.]. the methodologically best trial fails to generate a positive result. QED! thanks again.

  • Yep you are right prof Ernst, all the above studies are of no relevance, all the positive outcomes mean nothing as they are all prone to bias and have flaws etc. The only piece of research we need to pay any attention to is yours. Your 1 conclusion of “This is a polite and scientific way of saying that CST is bogus.” is all we need to know as your research doesn’t have any bias or flaws does it?

    • Fedup,

      Impressed as I am with your ability to post snippets of text that you feel shows that Edzard’s views are biased, it would be very helpful if you would provide some sort of link to the reports (or at least to the abstracts) so we can actually see the quality of the report.
      Simply posting ‘Conclusions’ from a trial does nothing to confirm the quality of the trial design – Afterall you wouldn’t want to exhibit bias in your argument.

      If you feel that Edzard’s views on the quality of research are wrong, perhaps you will accept the views of recent trials published on the National Council for Osteopathic Research (NCOR) website – surely if there are positive, well conducted trials into cranial osteopathy then they would appear here: http://www.ncor.org.uk/research/research-summaries/

      Well it seems not. The only part of their research section that lists Cranial research/studies is systematic reviews section. On the subject of methodological quality the cranial studies available state: (you’ll notice I’ve given you links!)

      http://onlinelibrary.wiley.com/doi/10.1002/14651858.CD004796.pub2/abstract#shortAbstract
      “The studies included in this meta-analysis were generally small and methodologically prone to bias, which makes it impossible to arrive at a definitive conclusion about the effectiveness of manipulative therapies for infantile colic”

      http://www.ncbi.nlm.nih.gov/pubmed/23131379
      “However, due to the current moderate methodological quality of the included studies, further research is needed.”

      http://www.ncor.org.uk/wp-content/uploads/2012/12/Jakel-cranial-SR.pdf
      “Because of the moderate methodological quality of the studies and scarcity of available data, further research into this area is needed.”

      These are all recent studies (within the last 2 years).
      They are not not studies chosen by Edzard or anybody else commenting here – they are chosen by NCOR!
      In each case the opinion of the quality of available research that of the researchers (some or whom are osteopaths)

      I have contacted both of NCOR and GOsC and neither of them have been able to provide details of any reliable research for cranial osteopathy for even a single condition. Both NCOR and the General Osteopathic Council have openly stated that there is a lack of good quality research into cranial osteopathy.

      So I guess we are can only draw 2 conclusions, either Edzard, NCOR, GOsC and the all researchers are wrong OR you are!

      SB

    • no all you need to pay attention to is the only reasonably sound study [not mine] and this fails to show a positive result.

  • Our son never slept more than 3 hours at a stretch until his 2nd birthday, where he fell asleep after a tractor ride at the local petting farm. He then slept through the night for the first time and continued to have a much better sleep pattern afterwards too.

    I trained as a tractor driver and opened the Infantulus Somnus Therapy clinic, and as a bonus mowed your lawn while treating your restless child.

    Or it may have just been a coincidence that he slept that night and I never bought my own tractor…

  • SB I have no problems with the articles or outcomes you have listed. but please read the conclusions I am happy with no definite conclusions more research needed, I think that is a perfectly acceptable outcome. they do not say “This is a polite and scientific way of saying that CST is bogus.” They do not emphasis that because of their research this form of therapy is bogus. They even state that some of their research is prone to certain types of bias. Cant argue with that. My whole argument has been about the flaws in prof Ernst review. If he is the only auhtor then his piece is as flawed as all the others, author bias etc, but for some reason his piece seems to be OK, flawless because hes done it.

    • if a group of therapists claim that this or that therapy is effective for this or that condition while there is no good evedence to show this to be true, IT IS BOGUS!!! what is so difficult to understand about that?

    • A word of advice, fedup: Don’t keep putting words into my mouth and don’t keep shifting the goal posts and expect me not to notice. Nowhere have I said that “no bias was present in his review”. (As EE himself said in his first response to you above, “no review can rule out author bias.”) This is about your contention that the research is “totally flawed”.

      You are at liberty to challenge the question, the methodology, the criteria and his evaluation of the findings; you are at liberty to suggest a better methodology, criteria etc, ones that would produce a more objective and reliable result, but you have elected not to do this. Instead, you repeat like a mantra your argument from personal incredulity that “the fact that prof Ernst is the only author” means the research is “totally flawed”.

      Presumably then, you think that if a team of six practitioners of CST conducted the review, it would be less flawed? Thank you for this demonstration of how you think. I’m sure we all better understand how you chose to become a quack in the first place.

      • Lol and you say I put words in your mouth. Ok does prof Ernst review follow cochrane protocols? No. Therefore as a systematic review it is open to flaws. Primarily author bias. On this basis his research is as likely to be as biased as a piece done by 6 cst practitioners. I would expect there piece to be full of bias unless they had seperate authors or editors that wernt cst practitioners. My problem with this whole article is that prof Ernst found 6 rtc and came up with a conclusion cst is bogus. Do you take that view? If you do you believe a flawed piece of research over 6 flawed pieces of research.

        • i estimate that less than 1% of all published systematic reviews follow the Cochrane protocol; i have done about a dozen Cochrane reviews, but i also see the benefit in doing reviews that are less tedious and take less than a year to publish. certainly, non-Cochrane reviews are not by definition not valid.

          • Ok I agree with you prof Ernst. But do you also agree that these other study’s are much more likely to be affected by bias? Intentional or not.

        • @Fedup:

          You have provided no evidence to back up your claims that Professor Ernst’s review is flawed or biased. To quote Christopher Hitchens, “that which can be asserted without evidence, can be dismissed without evidence.” We can therefore dismiss your assertions.

          If you want your assertions to be accepted (or even considered) you will need to provide better reasoning than what you have so far advanced, which amounts to nothing more than an ad hominem argument against Professor Ernst.

        • Thank you, fedup. When I end a sentence with question mark, that means it’s a question. It isn’t putting words in your mouth in the way you’ve been doing to me.

          “My problem with this whole article is that prof Ernst found 6 rtc and came up with a conclusion cst is bogus”.

          The truth at last.

          You don’t like the conclusion but you can’t actually fault the methodology, so instead you focus on the fact that there’s a single author. In your opinion, this is sufficient to dismiss the review as not just flawed but totally flawed.

          “Do you take that view? If you do you believe a flawed piece of research over 6 flawed pieces of research.”

          You haven’t shown any flaws in the research. Until you do, I have no reason to disbelieve it. Try reading the paper from the perspective of someone who, unlike you, has no vested interest in whether the review is flawed or not. (In anticipation of your protest, you have a vested interest in trying expose EE, who is the nemesis of all quacks, as someone who does “totally flawed” research). If, having read the selection criteria, analyses and discussion, you can spot something that is clearly wrong, the time to speak up is now. As EE said in his first response to you (which I’ve had to remind about three times now) bias should be demonstrated, not assumed.

    • Fedup,

      “but please read the conclusions” I did, including the part you chose NOT to post.

      Manipulative therapies for infantile colic. http://onlinelibrary.wiley.com/doi/10.1002/14651858.CD004796.pub2/abstract

      The section you did NOT want to post says:
      “However, most studies had a high risk of performance bias due to the fact that the assessors (parents) were not blind to who had received the intervention. When combining only those trials with a low risk of such performance bias, the results did not reach statistical significance. Further research is required where those assessing the treatment outcomes do not know whether or not the infant has received a manipulative therapy.”

      Maybe that is also why you chose not to post any links to the research!

      On 17th you chose to say “Again 6 authors for this study but hey, bias to high, right Prof.” The 6 authors you refer to recognise bias (something you clearly don’t) and they acknowledge that when only considering a low risk of bias the trial shows no benefit.

      You chose to quote this trial and you readily accuse others of bias. However you yourself displayed extreme bias in not posting the part of the conclusions that go against your argument.

      So regardless of whether you accept the ‘inconclusive’ results of not …. perhaps you could explain why you thought it best to cherry-pick the text, deliberately avoiding anything that did not support you views. (BIAS)

  • I found this and thought of you Prof ernst.

    Every quote or saying has a context that goes with it, and AbsenceOfEvidenceIsNotEvidenceOfAbsence is certainly no exception. It has to do with the flip-side of the all too frequently recurring case of assuming facts not in evidence. It’s no less safe to assume those non-evident facts are true than it is to assume they are false. It has to do with the cases where people weren’t aware of some possibility or piece of information and therefore jumped to the conclusion that it wasn’t there. If we don’t see or understand something, let’s at least consider that maybe our vision or thinking needs some adjusting before rushing to declare it completely absent or unworkable.
    It’s all too easy to jump to that wrong conclusion when it’s a case of blaming something other ourselves. It’s so much easier to point the finger at “O-O” or C++ or inheritance or parallel development, etc., than it is to point the finger in the other direction (just remember when you point your finger the other way, all the other fingers on that hand are pointing back at you 😉
    The context here has to do with lack of awareness. When people dismiss that possibility simply because they weren’t aware of it, or haven’t made the attempt to explore it, or haven’t looked beyond their own personal usage/viewpoint — then they are all too frequently jumping to conclusions based on lack of information rather than the presence of it. It may be that the conclusion was correct after all, but that doesn’t make it any less premature at that time, before it was adequately explored.

    • I found these quotes and thought of you, fedup.

      “One of the painful things about our time is that those who feel certainty are stupid, and those with any imagination and understanding are filled with doubt and indecision” (Russell)

      “Ignorance more frequently begets confidence than does knowledge” (Darwin)

      “The fool doth think he is wise, but the wise man knows himself to be a fool.” (Shakespeare)

      Now, what was your point?

      • Lol. I’m not certain of anything. I’m certainly not sure cst is bogus and implausable. Are you ? Prof Ernst seems to be. I imagine how it may work but I’m not sure. I’m on the fence but don’t like people who tell me that I must jump right. I think Iam ignorant about something’s but have more knowledge than u and prof Ernst on others. Though you are pretty confident everything I know or do is quackery. I must be a fool to try and offer an alternative to the blinkered brigade of skeptics.

  • being self-critical is what all scientists learn and most muster. have you learnt it? do you think you are critical about the claims you make regarding my “bias”?

  • Anyone,

    I’d be interested in seeing links to the research posted by Fedup – can you post please?

    Re: Cerebral Tissue Oxygenation Study. Having undertaken an intercalated degree in Child Health & Deterministic Chaos, and having undertaken a study looking at that RR interval analysis, I’m not sure what the clinical significance of a slightly warmer prefrontal cortex might be?

    Re: Visual Function study, not sure how the double-blind claim holds up given that the therapists weren’t blinded to the nature of the treatment.

    Just the first of many questions. From a personal point of view (12 years as GP, UK, Fulltime) I’ve had many patients asking about CST. I’ve always discouraged them on the basis of professional and evidence based knowledge, and I’d say anecdotally that the number benefiting from treatment has been no greater than one might expect from placebo. I see no reason to change my approach based on the discussions here.

    • Dr grimes I only mentioned those studies as it seems to be the general opinion here that cst cannot work as it is implausable and has no physical affect. These studies seem to show at least some physical changes. I will try and find these for you in the next couple of days and post links.

      • Great, thanks.

        In the interests of fostering polite debate, bear in mind that CST does sound implausible. Given the world is full of implausible things, we can’t rely on humans to be the best judge of efficacy. When it comes to CST, what I’m interested in is clinical efficacy – whether an intervention has a meaningful effect on a patient OVER AND ABOVE placebo. Like I’ve said, I have seen some patients (and their parents) who swear by CST, but flawed human psychology means I cannot rely on this as good evidence for recommending it to others. Researchers can be similarly flawed, so I cant even rely on a single trial. Cochrane reviews, of the sort described by and participated in by Edzard, are fantastically helpful in this regard.

        Ultimately, I will only be suggesting CST when robust trials exists showing benefit. None yet, and I’m not terribly hopeful.

        FWIW, keeping the temperature of these interesting and entertaining debates down is key, otherwise we’re just rolling around on the floor flinging poo at one another.

        Merry Xmas!

  • @fedup is it possible that the treatment is quackery? What would you accept as evidence of its falseness?

  • Justin. It may be quackery I’m not sure. But a study done by prof Ernst that is not only open to author bias but is only available if you pay to see it, conflict of interests, another pitfall, is not going to make up my mind. No wonder prof Ernst is pushing this subject and this review as “Wiley sells its publications, online and in print through multiple channels:”

    • It looks very much as if you haven’t even read the review that you are claimig is biased. If so, your opinion is worthless.

    • the fact that the article is not free is VERY regrettable; sadly, i cannot change this, and much of the medical literature is the same.
      the fact that you obviously have not read the article on which you have commented on in a dozen or so comments discloses you as a less than reputable commentator.

    • are you suggesting I have conflicts of interest? would you care to explain what they are?

  • has FEDUP gone into hiding? too much egg on the face?

  • Very interesting to read the blog post and comments. I trained as a CST therapist 3 years ago having reached middle age, stuck working in an office and wanting to do something more ‘meaningful’. I do not have a background in health sciences and so I picked an ‘accredited’ college offering a diploma and pathology training, and claiming to treat many medical conditions which certainly looked like a first class medical provider. You can imagine the shock I had when most of the way through the training at a personal cost of £5000 when I started reading some of the material posted by SB which unfortunately rang true and became a real threat to the college. What ensured was little more than lies and bullying tactics by the college tutors to keep the student body inline and away from debating the actual efficacy of the therapy, whilst they continued to treat babies and assess us on our ability to perform ‘energy drives’ using only our intention. I decided not to flush my sunk cost down the drain, kept quiet and slide out the door with my piece of paper. I practiced for a year afterwards, and whilst my experience definitely confirmed to me that CST for people in distress was beneficial due to human company and appropriate touch, the medical claims the college taught me and imparted enmass to fellow students who were more ignorant than me is just wrong, and could be plain dangerous. All measures should be taken to stop this practice, especially since the college made circa £100,000 from peddling incorrect information to our class. I’d be happy to post the actual sylabus and course notes to anyone who can help in this regard. Please just post your mailing address on subsequent comments. Btw this is a north London college based in east finchley, accredited by a voluntary body of other therapists who almost all have personal interest in maintaining the status quo.

    • Hi Sonny,

      Thanks for your comments. Pleased to hear that some of the info I posted resulted in students at the college actually having the courage to question what they were being told (and sold) – just a pity the tutors resorted to “bullying tactics” instead of providing the answers to your questions. Also interesting that it was seen as a threat to the college.

      Obviously I’d be very interested in seeing whatever info you have and also hearing more on your views of the actual training. Feel free to contact me on [email protected] and we can take if from there.

      Regards
      SB

  • The college in – North London.
    One aspect that is a clever financial business ruse is that this college run a Anatomy, Physiology and Pathology course with certification. The idea being that one does not need to go anywhere else to do APP if you are studying Cranio Sacral with this college.
    However, this course and certificate is not recognised at all anywhere outside of the college running the course. The certificate is therefore worthless. There have been a number of students caught out by this thinking that it was infact a bonafide recognised course. I remember one student who was also studying another alternative subject was told that to qualify they would need APP………….and was told that the one she had done with this North London college would not count …………she would therefore have to study and pay again for a proper course in APP somewhere else.

    Coming back to the discussion about Cranio…………a massive problem lies in the education of this system. What this North London college and other colleges that teach Cranio offer …………….is a one year intensive or a two year course ( both are part time )in Cranio Sacral Therapy. I do not believe that the teaching of this system is that good. It is a bit like a conveyer belt of paying students. Supposedly after one year or two years one becomes ” qualified “. And they can then make all these claims but with very little actual experience of ” treating ” or seeing clients. So in effect each year there are about 60 people that will ” qualify ” from a college. As for how good they will actually be is anyone’s guess. Supposedly they would need to be a certain standard.

    Is it worth having a session ? Well…………….I began the training and the college encourage you to have sessions yourself.
    Fair enough. I remember going to see someone that was qualified from this college and they were charging £70 for an hour. I thought the person and the experience was rubbish and I would not recommend them. However, I have also seen someone with many years experience and I thought it was fine for what it is. I would only consider paying £40 for an hour even with someone of experience…………
    Therefore like anything one learns……………..you seriously need experience out in the field of actually doing it on a regular basis. Nearly all who qualify simply do not have this experience and will never gain the experience. Perhaps out of the 60 who qualify from a college each year then maybe 3 of these people might begin to do it on a regular basis if they are lucky and that is an optimistic figure.

    Having a session when it is good and you are with someone you trust and have confidence in is ok………………….but then again having a session of Reiki can be ok too and so can a massage be ok. Did it ever cure me of anything……………….no. Was it a relaxing experience……………yes.

    I am an ex student who began the course but I could see early on that it was never going to go anywhere and I felt the education and qualification that goes with Cranio Sacral Therapy to be ” lacking ” and questionable. I decided to give it up.

    Doctors and G.P.’s usually study for a number of years to do what they do………….perhaps five years maybe more ?

    If something like Cranio had a 5 year study time then it would be a lot better for the students etc but that will never happen because it is about a quick turnover of students and finances. It is business. Many people go into this with the idea that they will study something they have an interest in and then when qualified they will have a new career that they want to do after a year or two. For approx 90 percent of all students this will never happen.

    For some who pursue the career angle…………………what becomes apparent is the idea of teaching the subject. A number of people therefore decide to become teachers. Teaching can provide some income along with clients …………some of whom will be their students.

    I, too, like Sonny can say a lot more about all of this but for now will leave it.

    But I do agree with the majority of posts and the blog.

  • I had CST one March. It worked. My baby was born 25th december. CST is good for making children without men.

  • Jenny said:

    I had CST one March. It worked. My baby was born 25th december. CST is good for making children without men.

    Wot?

  • Further to my post of 29 Dec 2012 regarding Cranio Sacral Therapy and the teaching / education etc.

    When I was in training I had experience of Cranio sessions from five full time different practitioners in total. I found that only one of these gave a good session with good technique along with good communication skills.
    The others…….on wishing to discuss……when asked about the session or on the subject of cranio itself did not appear to know what they were talking about or doing or simply did not wish to discuss. And their overall manner came across as irritable.
    I found that with the person who I thought was good……..experience and confidence in what one is doing generates a certain calmness and ease in general.

    A reason why some choose to teach comes down to finance. Earning a living from simply having clients is unlikely to generate much income and therefore the option of teaching becomes an attractive one. The teachers at a college gain from the experience because they earn some income from teaching, gain clients from the students themselves requiring sessions, and gain valuable practice and experience from being in the learning environment on a regular basis. It is infact a smart move. But as for how many get this opportunity……………..very few……………hardly any.

    Regarding trading standards and advertising flaws………………………..as soon as you call something a ” treatment ” or a ” therapy ” then it is implied that a healing is taking place. The overall claims that Cranio Sacral Therapy makes are astonishing. When one looks at the websites out there and the literature on these websites etc it is remarkable. Also I have not met anyone that can say CST helped them with any of the ailments that are suggested on these sites. So what the colleges are hinting at is that after one year or two years a ” qualified ” person can then do and claim something that fully qualified Doctors, G.P.’s and hospitals do not usually do which is claim to heal this or claim to heal that. And also charge above average significant amounts of money to do so.
    The best thing that these people could do in general is to look at the language they use to promote the system. That would be to drop the word ” therapy ” completely and also drop the word ” treatment “. And as for the ailments and claims of healing all these ailments………….drop all that …………once again use the language in a different way. Describe the system in a different way so that people are not given misleading information.

    So what does this leave ? Well in my experience of learning this system it is a hands on system whereby the ” practitioner ” gently places the hands on different parts of the body and from their training assesses a ” diagnosis ” of what can be happening according to the cranio sacral system. And from making contact with this system it is then allowed to express itself freely. And in allowing this to happen the cranio sacral system is somehow correcting itself accordingly. This then has a knock on effect with the body in general so that certain benefits result. This is why it can be noted that there is something natural or traditional or tribal or shamanic about the cranio sacral system.

    However, I have not met anyone that can diagnose anything specific. Or can cure anything specific.

    For example a ” diagnosis ” is only likely with someone good and experienced and even then it is likely that the good practitioner will describe certain things using language which can help point to something. Rather than directly coming out with a spot on analysis. And that is fair enough if client and practitioner can communicate freely with each other.
    And with a ” cure ” …………….if someone goes with a bad leg, or a particular illness then there is no certainty at all that Cranio will alleviate the symptoms of the bad leg or the particular illness.

    The premise with Cranio and what is behind the teachings is that Cranio works in ” layers “. So what is being suggested is that each session will offer something that will benefit the body as a whole. This will then have a knock on effect in helping to balance the body. So if someone goes with a bad leg it may take one session to help alleviate the symptoms or it may take six sessions or ten sessions or for whatever reason maybe the leg might not get better in the short term.

    And of course the major problem here is what if you go to see a Cranio practitoner for something specific………………….?? Are you prepared to keep on going for 6 months or a year ? Especially when there is no certainty of curing your specific problem.

    As an alternative subject and a genuine interest then the story behind Cranio Sacral Therapy is interesting and fascinating. Even learning some aspects of it is intriguing. However, as previously stated – the education of this system, the advertising, the trading standards, the claims and language used…………………..all of these things need looking at and correcting appropriately. And even then I think if anyone is to be reputable then it would probably take a good apprenticeship and training for many years. And that would probably mean one person with a keen interest willing to choose that path.

    More later when Sonny rejoins the chit chat.

    • Lets start with the positive aspects of cranio therapy. Firstly, it’s a very simple setup of two people in a room together with a table and two chairs. The (good) therapist listens intently to the client, shows empathy and then uses touch to help facilitate the “treatment” – often peace, relaxation, imparting a sense of safety or re-processing previous experiences. Framed in this manner, it is quite easy to see the benefits of the therapy and that it can be incredibly powerful for people in distress and crisis. The only shame is that society at large values personal encounters such as this at such a low level.

      However, once the words like “treatment” and “cure” and medical “conditions” enter the vocabulary of the therapist, serious and significant moral and ethical matters come forward. I don’t wish to debate the pros and cons here, but rather on this occasion raise some matters you can check out for yourself and come to your own conclusions.

      Lets take the College of Cranio Sacral Therapy as an example… their website http://www.ccst.co.uk begins setting the scene to members of the general public and prospective students as follows:

      “CCST is the most established college of Cranio-Sacral Therapy in Europe – currently in its 26th year.”

      And regarding who attends the training and why, “The college offers a wide variety of trainings to suit different needs – for professional therapists from other disciplines, for student therapists, and for those wishing to become therapists as a career change. Career Development Loans are available.”

      There is an article on the site written by the founder of CCST, Thomas Attlee, regarding “Undiagnosed sub-clinical meningitis” and the place that Cranio therapy has in the treatment of such matters. http://www.signifier.co.uk/CCST/articles2.html

      *** Read the article carefully, think about what impression it would have on members of the public considering such training, and how many good references are made to medical studies/trials to substanstiate claims in the article. *** Btw. Thomas charges £90 for half an hour treatments.

      As per CW’s previous points on teaching the subject to supplement therapy income, the following websites are for two college tutors:

      http://www.danielchapmancranio.com/what-is-craniosacral-therapy/can-craniosacral-therapy-help-me/
      http://www.doughutchings.com/phdi/p1.nsf/supppages/3956?opendocument&part=5

      Have a good look at the conditions listed, and what they may imply about cranio “treatments” to the general public when reading their practitioner websites. Given these two chaps are “registered” members of the Craniosacral Therapy Association (CSTA), tutors at the CCST and the CCST itself is a member school of the association, both websites are clearly in breach of the association’s own advertising guidelines and the CAP advertising code generally. Voluntary self-regulation in the therapy industry is a massive fail.

      I’ll leave the fact that students at the CCST are often suggested to have treatments with college tutors, at personal cost to the student and leaving a question mark to the impartiality of their subsequent assessment should they not choose to have ‘treatments’.

      PS. The diploma is printed out on an inkjet printer at the back of the class room, and is not recognised at any other educational institution.

  • Thanks Dr. Ernst for this wonderfull blog and also specificaly for this comment on CST. Being a physical therapist, I am often faced with dealing with both patients and fellow practicionners who believe in this kind of non-sense therapy and I always shake my head in despair to myself wondering when this folie will stop…

    I guess not for a while!

    BTW a good addidtion to your blog would be the twitter and facebook icons so we could easily spread your words of wisdom.

    Regards,

    Frédéric

  • Regardless of what some experience, my experience with this yesterday was wonderful. I was able to release a huge blockage that was causing all sorts of problems in my life. It was funny the way it happened, I was expanding my awareness throughout the day, and then was randomly invited to come try this for the first time! All for a reason 🙂

  • With a lot of interest I have read the review and the blog.
    As a patient, I remain very dubious about the practice of CST. I am a naturally sceptical person and ratio governs in my dealings with the health service.
    Nonetheless I rest in dubio because traditional medicine shows some major flaws which can put you, as a patient, in a very isolated and lonely spot.
    Traditional medicine is very good and controlled trials, testing of drugs, and in very specialised interventions (surgical etc). For these things, there is no better reference.
    Traditional medicine, however is not so good at systemic diseases that include large systems (central or autonomous nervous systems, immune systems, the brain …) and is therefore very much lacking in provinding cures or even sometimes alleviations of pain for diseases such as MS, AIDS, lupus, certain cancers, many types of auto-immune disorders, Alzheimers, dementia …
    GP’s will be the first to acknowledge that medicine has not gotten far yet. Yes, great progress has been made but in the end, a huge amount of disease or maybe just malaise is non-identified in cause and therefore incurable.

    Personally, I was diagnosed with ME, or CFS and fibromyalgia about a year post-partum.
    As a sceptic, I already had difficulty with a bogus diagnosis because literature indicated that these were not even diseases (although ME has a history of documented cases and is not merely a syndrome which the other two are).
    But no other diagnosis came, even though I tried (within my means) to exclude specific causes of my suddenly losing 90% of my mental and physical capacities.

    Many therapies were tried, most of them completely bogus and run by charlatans (with official titles, such as specialist in endocrinology and internal medicine, physical therapists,….) some therapists just acknowledged that they did not know how to help, did not know what the cause was, but that they were prepared to try and help me get through it (my GP, my psychiatrist, my massage therapist, my accupuncturist, my osteopath).
    Some of those not wanting to help were neurologists (“just sit it out, it will pass eventually”), my endocrinologist (“surely your endocrine hormones have nothing to do with it”).

    Those making claims that they would cure me, were all wearing official titles, some working in hospitals even, and others in spin-offs.

    For myself I can only say that the accupuncture and the massage brought short-term relief in the pain -more specific the tensing of the muscles- (not working longer than a week) and needed constant attention.
    Against the pain itself, only opioid painkillers helped.

    Psycho-analysis, massage and accupuncture and osteopathy all helped in the way that these were practitioners who took the time to do slow medicine: taking the time and the place (each one hour per session) to pay attention to you, listen to the problems (so you don’t have to burden your relatives) and giving you the time to formulate it.
    Because one of the largest problems you are confronted with in this type of disease that you lack the physical force to go and see a specialist (going there, waiting around…) but more importantly, you lack the mental focus to explain your problems clearly and articulately to a specialist when you are allotted 5 to 7 minutes.
    By the time I got into an office (because waiting seems to be essential), I just wanted to go home. I felt that I could in no way dialogue decently with the practitioner and make my case heard.
    The same happened when I was hospitalised and all my energy went to trying to understand the practitioner talking to me and forgetting what he said halfway through the sentence.

    My alternative practitioners and the psycho-analist never made me wait, and always took their time.
    So if one says that alternative medicine can be effective because they do this, then I agree. To say, that we should replace this by effective treatments that respect the same ‘principles’, then I must dissapoint you.
    In my health career, and that of my children, doctors that take their time and make you feel that they are there to help you form a small minority. My GP is a nice exception, as probably more GP’s are but then they admit they cannot do much for you except listen. Surely pressure of profit-making hospitals and profit-making doctors are to be blamed but there seems to be no space for slow, traditional medicine.

    Yes, my midwife gave me this service, but then most hospitals and doctors found me testifying of ignorance and dangerous behaviour to go into childbirth at home, only in the presence of two experienced midwifes.

    But why are we here? CST
    About a year and a half into my illness, I had regained a little strength, enough to let myself be driven to an osteopath that came recommended.
    I did a little research, had not much belief in it, it all sounded like bogus theory.

    I also researched my osteopath who trained as a registered physical therapist, manual therapist and osteopath. A total of 10 years study. This comforted me somewhat since I thought that if at least the osteopathy does not work, at least she understands human anatomy.

    My osteopath never claimed to cure me. She never claimed a diagnosis. One might object that this is an easy way out, but in the traditional medicine doctors I had met, the most trustworthy had done the same.
    She simply said that we would try and work together.

    The first time, I was extremely tired after the session. A few days after the first session, my stomach got very ill and I was hospitalised until 24 hours later, all that had been troubling my stomach had come out. Nothing ‘scientific’ came out of the tests. I had been experimenting a bit with vitamins and minerals the weeks before I started seeing the osteopath.
    It probably was a reaction to too much vitamins and other supplements being introduced at a time, but then I was anxious to get better.

    I went back to the osteopath every 3 weeks. I only followed psycho-analysis next to it and stopped all other treatments (a massage from time to time aside).
    Slowly, very very slowly, I picked up. Everytime I went to the osteopath, I felt worse for a few days and then it stabilised.
    I went for a year, every three weeks. By the third session, I managed to drive a car again and could take myself to my sessions.
    After a year, I felt a shift. I started to feel that inside me, strength was returning. It felt as if it was deep inside (I would say on a cellular level, it felt like every little cell getting stronger, but i have no evidence for this) that I was regaining strength.
    For the next 6 months, I kept very gradually getting better and got back to working 80%.

    I felt that this was the best I could do. I never thought progressing from there, having nearly lost my career and all of my identity in this disease.

    I started feeling the possibility of taking the risk of getting pregnant and I fell pregnant immediately. This proved to me that my body was getting a lot better for the body is self-protective. Also, i was 39 at the time, so getting pregnant at the first chance indicates positive factors.
    The pregnancy (and the continued osteopathy) brought on acceleration in my ‘regeneration’ process. Thanks to increased blood circulation (I can only assume) my health progress speeded up enormously to the satisfaction of my GP and my osteopath and my analist.
    The osteopath was able to correct problems with the stability of the bassin, and I went to her several times for acute problems of lumbago in the post partum period.
    After having given birth, I felt so healthy, that I stopped all treatments, happy to resume my life.

    I only went to the osteopath three times with acute lumbago, or numb leg and foot.
    The first times I visited a physical therapist first, and the problem only got worse.

    Mostly with one session, the osteopath can actually make the problem go away. Does she tackle the cause? I have no idea but I come out of session still sore, but relieved of the extreme pressure of the pain and a few days to maximum ten days later, my symptoms have mostly gone.

    My osteopath never makes anyone come more often than strictly needed.
    Patients is no problem. Getting a slot is difficult when you are a new patient.

    Sessions cost 30 pounds approx. which is extortion. On top of that the belgian health care system reimburses about 7.50 pounds of that.

    So, is it science? Definitely not the theory which they claim on several websites, or at least I don’t believe it.
    Is every osteopath competent? Definitely not, just like not all doctors are good and like not all medical colleges are good.

    Is it easy to do controlled trials? I think not, especially due to the difference of quality in practitioners, the not always clear diagnosis in patients and especially how to reach these patients in order to do trials because they are not in the eye of traditional medicine for they are wandering in a lonely fashion, keeping to look out for a cure.

    As someone said, i believe in the results: on a personal level, that is the only you can do. Because you look for a solution and you don’t care how you get to it. It is possible that my healing came about naturally without any influence from the outside.
    Personally, I think my cure came about the moment I accepted that I was ill, stopped fighting it and took enough time to go into my emotional and physical well-being. Working both on a mental level with my analist and on a physical level with accupuncture, massage and osteopathy for I can tell you : simply feeling the touch of a human being, done in an objective setting, is a large part of the cure for which traditional medicine does not take its recourse.
    Relatives cannot give you this.
    This, together with a very good discipline in eating and taking care of myself, stopping to push myself to get better too quickly, all contributed to my getting better.

    But only of my osteopathic treatment can I say that I felt it had concrete impact which made me feel the progress in clicks rather than in gradual progress.

    I would be happy for there to more research into how it works, if it is really working, but in the meantime, I have no choice, when my body aches, I first turn there and only if my osteopath cannot cure my mechanical problems (I would not go for ear infections 😉 or for smear tests or other absurdities) would I see someone else.

    It is only an anecdote, in my case it has cost me two and a half years of my life.

    So many people have vague diseases these days, that one wonders if traditional medicine cannot do a little bit more research to help those whose quality of life is severely affected, mostly women who have stepped out of active life because of a debilitating but not (directly) fatal condition.
    I even know of several doctors who are searching in alternative medicine, looking out the right osteopath, doing strange natural remedy tests, looking for a cure for their wifes. Even they separate the personal quest and the scientific option for a better future.

    Yes, we need research but in the meantime people look for relief or for (partial ) cures.

    It got to be a longer post than intended, I have never written about this episode (now 1 year since my childbirth and about 1,5 year after my last symptoms) and still building up muscle everyday, not back to my original strength and stamina but then it would be difficult to match 😉

    I hope to have contributed something, even though it is not scientific.

    I.
    Brussels

  • Iris…………….a long post indeed. However, your discussion appears to be about an osteopath. This particular page concerns Cranio Sacral Therapy. You have not actually specified whether you have experienced Cranio Sacral Therapy or not and if so to what extent.

  • I´ve been working as a physiotherapist in Düsseldorf / Germany since 1992 and I hold a master degree in biology (University of Kiel / Germany). I´m also a approved health-practitioner (physiotherapy).
    Between 1998 and 2008 I made three trainings with lot of classes (and lot of money) in CST within the Upledger Institute Germany and USA, Osho Uta Academy Cologne and Karuna Institute Devon (Franklyn Sills) with more than 1.500 hours. Let´s say I was deeply involved with CST and had some amazing experiences as a pupil and as a practitioner. But I abandoned CST completely. Why?- Because I´m nowadays convinced that CST is fake. There is no CSR and none of the so called CS-Patterns of the skull or body exists. I was trapped in a kind of trance and hallucination because of the fascination I had but it´s all gone. When I put myself into trance (and I´m very skilled in that, too) I can feel and detect a lot of things. But without trance I can only feel the skin, the temperature, muscle tonus and so on. That´s it. There is nothing more to say.

  • Intriguing that no practitioners have posted that use CST effectively in their practice. It is widely accepted by Osteopaths around the world that this therapy is entirely valid and that extensive research exists to demonstrate that a rhythmic motion of the cranial bones exists in many species including humans. The purpose is to distribute cerebro-spinal fluid and reduced range of motion in cranial bones results in less availability of nutrients for the brain.
    Readers should check this link to see real evidence of this physiology: http://www.cranialacademy.com/researchBONE.html

    • @PKH

      There are a lot of references on that page you linked to. Could you pick out the one that you believe is most convincing and provide a link to the original paper?

      • Ballard RE, Wilson M, Hargens AR, et al. Noninvasive measurement of intracranial volume and pressure using ultrasound. American Institute of Aeronautics and Astronautics Life Sciences and Space Medicine Conference. Book of Abstracts, pp. 76-77, Houston, TX, 3-6 March 1996.
        Ueno T, Ballard RE, Cantrell JH, et al. Noninvasive estimation of pulsatile intracranial pressure using ultrasound. NASA Technical Memorandum 112195. 1996.
        Ueno T, Ballard RE, Shuer LM, Yost WT, Cantrell, Hargens AR. Noninvasive measurement of pulsatile intracranial pressure using ultrasound. Acta Neurochir. 1998;[Suppl]71:66-69.
        Ueno T, Ballard RE, Macias BR, et al. Cranial diameter pulsation measured by non-invasive ultrasound decrease with tilt. Aviation, Space and Environmental Medicine. 2003;74(8):882-885.

        • are you sure that this has anything to do with cranio-sacral therapy?
          here is the abstract:
          INTRODUCTION:

          Intracranial pressure (ICP) may play a significant role in physiological responses to microgravity by contributing to the nausea associated with microgravity exposure. However, effects of altered gravity on ICP in astronauts have not been investigated, primarily due to the invasiveness of currently available techniques. We have developed an ultrasonic device that monitors changes in cranial diameter pulsation non-invasively so that we can evaluate ICP dynamics in astronauts during spaceflight. This study was designed to demonstrate the feasibility of our ultrasound technique under the physiological condition in which ICP dynamics are changed due to altered gravitational force.

          METHODS:

          Six healthy volunteers were placed at 60 degrees head-up, 30 degrees headup, supine, and 15 degrees head-down positions for 3 min at each angle. We measured arterial blood pressure (ABP) with a finger pressure cuff, and cranial diameter pulsation with a pulsed phase lock loop device (PPLL).

          RESULTS:

          Analysis of covariance demonstrated that amplitudes of cranial diameter pulsations were significantly altered with the angle of tilt (p < 0.001). The 95% confidence interval for linear regression coefficients of the cranial diameter pulsation amplitudes with tilt angle was 0.862 to 0.968. However, ABP amplitudes did not show this relationship. DISCUSSION: Our noninvasive ultrasonic technique reveals that the amplitude of cranial diameter pulsation decreases as a function of tilt angle, suggesting that ICP pulsation follows the same relationship. It is demonstrated that the PPLL device has a sufficient sensitivity to detect changes non-invasively in ICP pulsation caused by altered gravity.

  • Then how come it doesn’t work better than placebo? And how come neurosurgeons never observe this “motion”

    The rythmic motions are either or both, the patients or the observers own pulse or simply an ideomotoric imagination.

    More credible reading:
    http://edzardernst.com/2012/12/up-the-garden-path-craniosacral-therapy/
    http://www.sciencebasedmedicine.org/cranial-manipulation-and-tooth-fairy-science/
    http://www.sciencebasedmedicine.org/alas-poor-craniosacral/
    http://www.skepdic.com/craniosacral.html

  • Moskalenko YE, Kravchenko TI, Gaidar BV, et al. Periodic mobility of cranial bones in humans. Human Physiology. 1999;25(1):51-58.
    Moskalenko YE, Frymann VM, Weinstein GB, et al. Slow rhythmic oscillations within the human cranium: phenomenology, origin, and informational significance. Human Physiology. 2001;27(2):171-178.

    These studies demonstrated that the medical claim that that cranial sutures ossify in the adult human is false. This was confirmed by histology of the cranial sutures:
    Retzlaff, E W, Mitchell F L and Upledger J E 1977. Sutural Collagenous Bundles and their Innervation in Saimuri Sciureus. Anatomy Records 187:692.

    You can also view cranial changes in this 16 year old girl treated by myself and a Functional Orthodontist over the course of 16 months.
    https://www.dropbox.com/s/tmo6su1v104f3zy/cranial%20changes.jpg?dl=0

    • What do you think those two x-rays show and what is the clinical significance?

      • I think they show how very poor evidence can be acceptable for crazy people.

      • If you take a careful look at the measurements you will see the overall change in the dimensions of the cranium which is not a normal result over a year in a teenager. They can only result from her treatment by cranio-sacral therapy and functional orthodontics.

        • Well, I can see two x-rays that have lines in different places and different numbers on them and a title that says ‘Osteopathic treatment during orthodontics’. You imply they are the same person, but there are no IDs on them. There are no dates on them either. You say the dimensional changes are ‘not normal’ for a teenager over a year, but don’t substantiate that. You haven’t said why you believe those changes are beneficial. You say the changes ‘can only result from her (unspecified) treatment of ‘cranio-sacral therapy and functional orthodontics’, yet give no reasoning or evidence for any of that or that they are causal. You’ve said nothing of any possible harms.

          Can you see why someone might be somewhat skeptical of your claims?

        • I see two quite different X-ray’s which may well be of the same subject at different times but very clearly taken with different equipment and at different angles. E.g. the cranium is not in the same position relative to the spine which may explain the rather small numerical differences measured. There might even be a difference in focal length between the images. This raises serious suspicions as to the validity of drawing any conclusions from these measurements.
          Further, who is to say that normal, unrestrained growth of the headbones does not happen with some degree of asymmetry??
          Until you provide references to the description and validation of this measuring technique and its validity in demonstrating the influence of CST or similar caressing, my suspicion remains that this demonstration is pure and simple make-believe.

          I have studied the peculiar history of Craniosacral “therapy” (which is a popular pseudohealth service in my home country) In my training years I performed and participated in numerous CNS procedures (as well as necropsies).
          Applying a skull fixation device, drilling and sawing up many a skull and then observing the living, soft tissues within it, would without doubt have made one confidently aware of any possible movement between the cranial bones or any movement or pulsation other than that obviously caused by pulsating circulation.
          I can confidently assure the audience that the skull bones are securely and immovably joined in adulthood and the concept of craniosacral “movements” or pulsations is demonstrable nonsense.

          • Indeed, Björn. I do hope PKH will come back and provide answers to our questions.

          • The X rays were taken from file and had never been submitted in a journal article so relevant dates were not added at the time they were taken. The second x ray was asked for by the mother to demonstrate that changes had taken place as it is not the policy of the dentist to perform multiple x rays on young subjects.
            As I was not the operator of the equipment I cannot comment on whether the angles were fixed but I am sure a head brace is used.
            “I can confidently assure the audience that the skull bones are securely and immovably joined in adulthood”
            You have obviously not read the following study which demonstrated cranial sutures contain blood vessels nerves and connective tissue anchored by Sharpey’s fibres.
            Retzlaff, E W, Mitchell F L and Upledger J E 1977. Sutural Collagenous Bundles and their Innervation in Saimuri Sciureus. Anatomy Records 187:692.
            Ossification of cranial sutures is a rare occurrence and not the norm as stated in medical training.
            As we are clinicians and not researchers it is patient feedback that demonstrates the efficacy of our techniques. If they were ineffective we would not be successful in our practices. The opposite is true as our waiting lists show us.

          • “I can confidently assure the audience that the skull bones are securely and immovably joined in adulthood”

            Bjorn – anyone with basic palpation skills knows that’s not true. You should have left that part out.

          • @PKH

            So many questions raised by your last comment, but, essentially, there is absolutely no way of verifying any of the details you have given.

            How much do the sutures move?

  • “I can confidently assure the audience that the skull bones are securely and immovably joined in adulthood”
    Bjorn – anyone with basic palpation skills knows that’s not true. You should have left that part out.

    Yet again someone calling him/herself “jm” wheels out another barrel of tiresome nonsense. Probably thinks that because (s)he can move the soft tissues covering the cranium about with his/her fingers that (s)he has proven that you can move the cranial bones in relation to one another. Just ask any neurosurgeon or pathologist how completely fixed and immovable the cranial bones are.

    PKH pulls out a reference to a 1977 article about how Upledger himself, the inventor of modern cranio-nonsense, and his gang found living tissue in the cranial sutures of squirrel monkeys! Does (s)he think we are stupid??
    Of course they found living tissue. They are trying to prove their moneymaking make-believe.
    Our whole body (and the squirrel monkeys’ too) is made of living tissue with vessels and nerves. Nothing new there. Upledger was a charlatan who made up this particular brand of petting and caressing out of the fantasy of another imbecile who thought a bone in our head resembled the gills of fish(!!) and therefore he started imagining he felt “respiratory” movements.
    Upledger took this balderdash and started a quackery business on it.

    And as to PKH’s reply…
    We are not talking about ossification of the sutures. In high school biology class one can learn that there is connective tissue in the cranial sutures but it is firm and the sutures so complex, narrow and tight that there is no chance of any palpable movement if any at all. Now jm and PKH go and make yourselves useful somewhere and stop bothering the audience with your display of delusions.

    And a word to PKH…

    As we are clinicians and not researchers it is patient feedback that demonstrates the efficacy of our techniques. If they were ineffective we would not be successful in our practices…

    Try substituting toenail massage for your usual performance. If you change nothing else, you will find that you will be just as “successful”. The practice of CST is another glaring example of theatrical placebo based on nonexistent effects. It does not matter what or where or how you poke and prod. If you make the seductive claims and reassuring bunkum, you will procure the same results.

    • “Just ask any neurosurgeon or pathologist how completely fixed and immovable the cranial bones are.”

      I have. They move.

    • Björn, each time “jm” and others lend support to a branch of alt-med they provide increasing evidence for that branch being both quackery and health fraud.

      • “Pete 628” – the fact that cranial bones are not immovably joined isn’t a branch of alt-med. It’s simply anatomy. And for Bjorn’s fixating, drilling and sawing…it probably doesn’t matter that he thinks they are immovable. But his earlier comment would have been better without the last two paragraphs. Relax.

        • @jm

          PKH never answered; maybe you can?

          How much do the sutures move?

          • Maybe PKH didn’t think you were asking seriously. Why would that be, do you think?

          • @jm

            Why? Don’t you think PKH knows the answer or don’t you think he/she is inquisitive enough to have ever found out?

          • @Alan – Since you’re in the mood for speculating…tell me, what practical application would there be for measuring cranial bone articulation? Say someone actually spent the time, money and effort – what would a practioner of any kind do with such information?

          • LOL

            It would seem – to the curious anyway – that if a cranio sacral therapist believes he/she can create movement, then it might be both interesting and useful (or maybe even essential) to know how much they move, what the maximum movement is (or, ideally, the relationship between force applied and distance moved) and how movement is related to health benefits. But perhaps no cranio sacral therapist has ever been that inquisitive.

          • @ Alan – Yup, seems like relationships would be useful/essential. I don’t see how measurements would help. Not until everyone is the same size, shape, condition, constitution, etc. Until then, I guess you’re stuck with hands on learning, from an experienced practioner.

            Then again, maybe Upledger sells tiny little rulers that practioners use. Who knows?

          • You’re lack of curiosity is…curious. But maybe PKH will return to answer the question.

          • While we wait, I’d love to hear about your personal experiences with CST. I’ve never had an Upledger style CST session – the only cranial work I’ve experienced were more osteopathic type manipulations to treat some injuries (from what I hear, Upledger uses minimal amounts of pressure, osteopathic significantly more) . What was your impression of the work?

          • @jm

            Why on earth would you think I’ve ever had CST? But of course, you don’t…

          • I was hoping you were sincere when speaking about curiosity. Wouldn’t have bet on it…but one can hope. Oh well, it was worth a shot.

          • LOL (as you’re fond of saying)! That’s the funniest thing I’ve read today…but the day is young here.

        • We are always learning.
          By prodding and annoying our altmed-afficionado’s with truth and reason we learn more and more about them.
          I think we can now confidently assume that “jm” has vested interests in the practice of CST. He/she is most probably caressing people’s craniums and behinds for money and self-fulfilment.

          The fictive concept of movable bones in the cranium and pulsating spinal fluid that is thought to be felt through the cranium and the sacrum is what CST is all about. The clients are told that the practitioner can diagnose by feeling the pulsating movements through the skull and spine. With light fondling of the areas around the CNS, the practitioner purports to correct the problems found.

          Likely?… No, I don’t think so either.

          The peculiar thing is that the practitioner believes ever so intensely in this obvious nonsense. Sometimes they are of course simply feeling their own pulse but the reason they think they are “feeling” other, slower or faster, not pulse-related movements of the bones is pure imagination or, as it is called in psychology, the ideomotor phenomenon
          When the practitioners enjoy the fulfilment of being considered powerful healers by clients who are grateful to anyone who provides TLC in ample doses, and even make money on top of that, their folly is fixed for good as a result of cumulative irrationality.

          “jm”‘stuttering about us being wrong is the symptom of his/her dissonant mind desperately trying not to let an inconvenient truth get in the way of his/her playing doctor for money.

          “jm” is not trying to convince me or Edzard or Alan or Pete 628 or anyone coming here for information on CST. (S)he is desperately trying to convince her-/himself.

          • Wow Bjorn, another confident assumption that missed the mark. I have no interest in, and don’t practice, CST. Anywho, I actually have talked to a few neurosurgeons about cranial fixation/movement. Have you? If not, you really should. Especially one with 30+ years of experience. It’s quite entertaining, and informative.

            There was no mention of Harry Potter, tooth fairies, or the sun being made of coal. Quite a bit about common sense disappearing from medicine, though.

          • How interesting.
            Maybe we could have the names and addresses of these specialists. If they have new important, breakthrough information it should be studied and the results published.

            BTW… I have spoken to several young people who have seen Santa Claus.

          • New important breakthrough information? It’s common knowledge that common sense is disappearing from medicine. An interesting study would be looking at your fictional references when people call you on your bullshit. (plenty of data on the gua sha thread…)

  • The cranial deflection induced by the force that practitioners claim to use is in the order of 50 nanometres: one tenth of the wavelength of light; less than a thousandth of the thickness of a human hair.

    Is this level of deflection detectable by the practitioner? No. Is it useful to the patient? No. Some patients find the treatment soothing and it likely soothes the practitioner’s ego.

    The cranial rhythmic impulse (CRI) that practitioners claim to be able to detect does not exist outside of their vivid imagination and wishful thinking. Teaching craniosacral therapy is a conjuring trick that works by… (Sorry, that’s a trade secret!)

    Rather than provide endless references that quacks never bother to read, I shall end with a thought based on simple anatomy and basic mechanics. If the tiny deflections induced by craniosacral therapy produced health-influencing physiological changes then the following would be extremely dangerous: riding a roller coaster; travelling in a car, bus, etc.; thunder and loud music; moving one’s head around; eating (jaw muscles deflecting the cranium).

    • …not to mention walking into a quack’s office!

      • The prob is that : quack seems immunized to common sense. It’s pretty obvious that no “CRI” exist, every damn move would give you a headache. But it’s not counting on “blablablah medic don’t know, I know better even if I’ve never seen a skull of my entire life and I’m the most clueless on this planet”. I know a lot of neurobiologist in my institute, no one ever heard something that stupid (and they aren’t all M.D !).

    • Pete 628 said:

      The cranial deflection induced by the force that practitioners claim to use is in the order of 50 nanometres: one tenth of the wavelength of light; less than a thousandth of the thickness of a human hair.

      And approximately one ten-thousandths of the distance between two ridges of a fingerprint.

      • That would be a bit different than “securely and immovably joined”.

        • @jm

          So, do you think 50 nm is clinically significant and knowingly achievable (transiently, at least) by a CST practitioner?

          • The only “securely and immovably joined” objects in the known universe that we currently have a plethora evidence of is the minds of quacks.

          • Do you really think you need that amount of force to clinically affect the body? If you’re as curious as you say you are, do your own experiment. Watch what happens to someone when you say “relax your shoulders”. Try it with a bunch of people, and pay attention to what happens. You really don’t have to practice much to have a duplicatable, knowingly achievalbe, clinically significant affect – with no force at all.

            If I’m remembering right, CST practioners use the amount of pressure equivalent to a nickel, or quarter, or something like that. I think that’s the amount of force “Pete 628” is referring to. If the CST goal was to solely manipulate cranial bones…that amount of force certainly seems implausible. But that’s not what they’re after. Manually manipulating the relationship of the bones themselves takes quite a bit more force.

            You could also just ignore the structure and hack your way through – like Snow in Bjorn’s video. (next time put up a spoiler warning, Bjorn…haven’t seen that season yet).

            Pay attention to the structure and skillfully manipulate, or hack through – whatever works for you. But if you are ignorant of the fact that cranial sutures are articulate joints…hacking will probably be the only tool in your box.

          • Articulate joints??? Well that’s what I call stretching definitions. 🙂
            We are talking about adult skulls here.
            Believing that the position of the cranial bones or the flow/pressure of the liquid underneath can be detected/affected by light fondling (with the pressure comparable to the weight of a small coin) is plain stupid.

            I can’t believe I just wrote another longish reply to feed someones trollish fervor. Must be the “Man-flu” I am nourishing right now. I have been relaxing my shoulders for all of an hour. According to “jm” it should work wonders but I may be one of those not amenable to therapy for the common cold? … OK, I struck most of what I wrote. I hereby promise to stop caring about thick hea… eh, skulls. Now where is that Paracetamol?

          • @jm said:

            If I’m remembering right, CST practioners use the amount of pressure equivalent to a nickel, or quarter, or something like that.

            Well, that sort of puts a number on the applied pressure. But if someone has measured the pressure/force a CST practitioner exerts on the cranium, have they also measured the physical effect of that force?

        • That would be a bit different than “securely and immovably joined”.

          “jm” got his/her restitution at last! 😀

          “A little bit goes a long way” but I seriously doubt that this sub-microscopic “bit” that made jm so happy is enough for even the most CST-sensitive fingertips to discern.
          Enough of this nonsense.
          Let’s instead have a look at one of the few practical ways to dispace the cranial bones. (watch till the end please 😉 )

  • I did wonders for my allergies. WONDERS! I HIGHLY recommend it.

    • Cindy said:

      I did wonders for my allergies. WONDERS! I HIGHLY recommend it.

      The problem is you will be unable to offer even a jot of good evidence that it did.

      • Why would that be a problem?

        • jm said:

          Why would that be a problem?

          Oh it’s not really a problem. Unless you care about misleading or deceiving people and experimenting on them, that is.

          • @Alan
            What makes you think Cindy is experimenting on people? That’s quite a leap, based on her comment. Do you have proof?

          • jm

            You’re misunderstanding. This isn’t about Cindy’s personal anecdote, but the generalisability of taking what she said somehow as evidence for effectiveness for others. If there is no good evidence, then others are experimenting on people by not having a clue whether what they are doing is effective or harmful.

          • Alan

            Cindy is very clear that she’s relaying personal experience. There’s no indication that she’s a practioner, making a professional treatment recommendation. There isn’t even a hint that she’s offering her experience up as clinical evidence.

            Let’s assume that you just misunderstood, and are not trying to be deceptive or misleading, by making Cindy’s comment into something it’s not.

  • jm

    Yes it is very clear she’s referring to her personal experience. However, Prof Ernst’s website is not a health equivalent of Trip Advisor where people give their personal views on whatever quackery they’ve just had. The point I was making, although entirely lost on you it seems, is that an anecdote such as that tells us nothing about the efficacy of CST (which, after all, is what this post and blog are all about) because it is just that: an unverified and unverifiable anecdote and is not a basis on which any healthcare decisions can be made for others.

    • Alan

      You honestly think that people are making healthcare decisions based on a blog comment? Really? Are you sure?

      • jm

        You’re very close to getting a ‘good grief’. If you can’t see the point I was making, then I can help you no more.

        • Alan

          Oh no! Not a ‘good grief”! Make whatever point you want, but I’m sure everyone knows that people commenting on their personal experience realise it’s not clinical evidence. And nobody makes decisions on healthcare based on a blog. Or blog comments.

          I’d even be willing to bet that for the majority of people, clinical evidence doesn’t come into play (and rightly so) when making personal healthcare decisions. But, good grief away! You could even add in a LOL if it makes you feel better!

          • @jm

            I’ve made my point several times now and I’m not sure what would be the benefit in saying it yet again.

          • clinical evidence doesn’t come into play (and rightly so) when making personal healthcare decisions

            It’s those parentheses that are daft, jm. Does a jury member think that evidence doesn’t come into play when making decisions as to a person’s guilt. You’re glorifying ignorance: that’s the real problem.

          • Franko

            Actually, it’s the other way around. Learning how to listen to your own body’s responses to physical medicine (like CST) would be the opposite of ignorance.

            Your jury analogy isn’t relevant to this particular situation.

          • jm
            “Learning how to listen to your own body’s responses to physical medicine…” Ah, here we go! Plea to subjective experience. But the unreliability of our subjective judgements makes us the least well placed to judge our own bodies. It’s the inner conviction we know what’s wrong with ourselves and what’s putting it right that underpins the widespread belief in nonsense.

          • Franko

            Perhaps your inner conviction is allowing you to believe that nonsense? For whatever reason, you’re a bit confused, or being misled.

          • @jm,
            I know i’ve said that I will ignore you, however, your ignorance is too great to be ignored.
            ~
            “Perhaps your inner conviction is allowing you to believe that nonsense? For whatever reason, you’re a bit confused, or being misled.”
            On reading this thread, the only confused things I see are your “arguments”; the lack of rationality is breathtaking.
            ~
            This gem is as “good” as it gets from you;
            “I’d even be willing to bet that for the majority of people, clinical evidence doesn’t come into play (and rightly so) when making personal healthcare decisions.”
            I’m not sure what you are advocating (because of your confused and nonsensical statements), but it seems as though you suggest that people should decide their own healthcare based on their “feelings”, and that clinical tests ate pointless because people just don’t need them? Does the idea of it being a “personal” choice mean they can do whatever they want, such as refuse blood transfusions for themselves and their children, take herbal brews for diseases like HIV, or treat pneumonia with green tea?
            ~
            Your interchange with Alan Henness shows a level of abstruseness and lack of logic. You concocted an argument on the flimsiest premise and I’m surprised Alan even bothered to respond to the nonsense.
            ~
            As for this;
            “That would be a bit different than “securely and immovably joined”.”
            Do you understand how small microns are? The force of a “nickel” will cause the minutest of deflections, yet you claim that it has some therapeutic benefit? Do you have any understanding of the basic laws of physics?
            ~
            I am not sure whether you are deranged or a seriously noxious troll. Either way, I am also surprised that Edzard allows your blatherings on here. Maybe, the purpose is to illustrate the absurdities of the mind of the alt-med?
            ~
            My question for you stands; in which branch of alt-med are you involved?

          • Frank

            I don’t think you realize how much ‘extra’ you read into comments because of your fundamentalism. You should have a disinterested third party read your comments on various posts and point it out to you. You could also have them add up your logical fallacies. While they’re at it, ask if they would classify you as deranged or a seriously noxious troll. You might be surprised.

            While you’re waiting, you can contemplate the nature and function of a structure that has the ability to articulate (no matter how small) vs one that is solidly fused.

      • @jm,
        I was going to ignore this but I need a good laugh, so here goes;

        “I don’t think you realize how much ‘extra’ you read into comments because of your fundamentalism.”
        Hmmm, is “fundamentalism” a belief that science will answer most of the questions, then yes.
        ~
        You should have a disinterested third party read your comments on various posts and point it out to you. You could also have them add up your logical fallacies. While they’re at it, ask if they would classify you as deranged or a seriously noxious troll. You might be surprised.
        ~
        “a disinterested third party”? Where would I find one of those? As soon as I found someone and explained the situation, they would cease to be a …………….
        Nonetheless, I’ll humour you and I’ll throw it open to readers of this blog; does anyone think I am “deranged or a seriously noxious troll”? All comments welcome!
        ~
        Now that you think you’ve made a significant point, lets take up where we left off;
        When will you answer my questions, and,
        When will you reveal what branch of alt-med you practise?

        While you’re waiting, you can contemplate the nature and function of a structure that has the ability to articulate (no matter how small) vs one that is solidly fused.

  • I recently had this recommended for treating my daughter’s chronic headaches. I had never heard of it before (there is way too much woo out there to keep track of). This sounded no different (fundamentally) to another “treatment” that was recommended to us. That treatment involved flopping my daughter’s hand while asking a question. Fortunately, my wife saw that it was rubbish. Unfortunately, only after wasting money on it. Hesitant to leap into the latest suggestion, I put off CST. Glad I did. A recent mention of it on the SGU prompted me to look it up, which led me here (from WikiPedia). Just that fact that it’s featured here verified that my bullshit detector didn’t steer me wrong in this instance. A great read that cost me time (well spent) and saved my wallet from being drained.

  • Thanks for writing this article. I was glad to find a skeptical take on CST as there is little material out there to weigh the practice against, especially with this level of rigor. Even finding an explanation for how it works is a chore.

    I completed one session of CST at the recommendation of a relative not known for looking for counterfactuals. During it the practitioner held the weight of my held for about 45 minutes moving it gently in various directions, then I recall he put his hands under my pelvis/sacrum and then briefly pushed through my feet from the soles, which, I think, was to see it the force would translate through my legs, pelvis, torso, neck, to move my head. I think this is to check for muscle tension. The sensations during the session were pleasant. I think my neck and shoulder muscles gradually relaxed and when they did he would move my head from side to side or turn it. Perhaps, this was also checking for resistance. This went on for a while and I noticed my breathing becoming shallower as I relaxed and it felt very much like a meditative state. After the session my neck and shoulders felt easier to move. The next day there was some soreness in my neck and down into both my arms, but overall I felt fine.

    My pet theory is that holding my head allowed the muscles in my neck and shoulders to relax which decreased the pressure on my nerves in those areas and caused cascading freer movement (within limits) and increased the flow of various bodily fluids: blood, lymphatic, csf(?). It was kind of like the short-term increase of range of movement after stretching session without the struggle against resisting muscles.

    I don’t know if it could lead to longer lasting changes in chronic muscle tension. I don’t know if the explanations the practioners give are the strongest argument for the practice. I would not take Deepak Chopra’s explanation of quantum mechanics as the best argument for it, pillory it, and claim QM is all bunk. Unfortunately it does seem like there is claim-creep happening to exaggerate the effects and attract more customers desperate for relief from whatever intractable conditions they possess. There may be a small but valuable kernel of truth in there somewhere, which would be a shame to discard with the chaff that accompanies many “treatments” that turn out to be a sham. All science started out as bad explanations gradually refined or discarded through time and testing. It’s a process worth remembering before we leap to all or nothing thinking that I know I am certainly prone to.

    The cost to run a test for yourself is rather cheap. No more than a massage it seems. The worst outcome I’ve had so far is relaxation, much like a massage.

Leave a Reply to PKH Cancel reply

Your email address will not be published. Required fields are marked *

This site uses Akismet to reduce spam. Learn how your comment data is processed.

Subscribe via email

Enter your email address to receive notifications of new blog posts by email.

Recent Comments

Note that comments can be edited for up to five minutes after they are first submitted but you must tick the box: “Save my name, email, and website in this browser for the next time I comment.”

The most recent comments from all posts can be seen here.

Archives
Categories